cual es el valor absoluto de -x+2

Answers

Answer 1

The absolute value of -x+2 is 2-|x-2|.

What is the Absolute Value ?

The non-negative value of a real number x without regard to its sign is known as its absolute value or modulus in mathematics and is indicated by the symbol |x|.

Since the absolute value is isolated, we set it equal to the positive and negative of the constant

Therefore,

|(-x+2)| =
for x < 2, -(-x + 2)


for x ≥ 2, (x + 2)

Hence, This is so that no matter the value of x, the value of the function is always greater or equal to 0.

To learn more about Absolute value click on the linkhttps://brainly.com/question/24368848
#SPJ10


Related Questions

1 3/4 x 4 2/6 nnnnnnnnnnn

Answers

The result of the multiplication operation yields 7 7/12

Multiplication of numbers

From the question, we are to multiply the given fractions. The given fractions are

1 3/4 and 4 2/6

First, we will convert the fractions to improper fractions

Converting 1 3/4 to an improper fraction

1 3/4 = 7/4

Converting 4 2/6 to an improper fraction

4 2/6 = 26/6

Thus,

The multiplication operation becomes

7/4 × 26/6

= 7/4 × 13/3

= 91/12

= 7 7/12

Hence, the product of the given numbers is 7 1/12

Learn more on Multiplication of numbers here: https://brainly.com/question/858368

#SPJ1

2. A different pool had an area that is of the form
▢ × 102 + ▢ × 101 + 6
and that can be written in the form x3 ,
where x is a whole number.

A) Decide what your number could be.



B) What is the perfect square number that is closest to the number you chose? What would the side length of a square pool with that area be?


C) Estimate the side length of a square pool with the area you chose in part a).

Answers

The area of the pool given by the expression, ∆ × 102 + ∆ × 101 + 6, gives;

A) The whole number x = 33

B) The perfect square number closest to 33 is 36

The side length of a square pool with an area of 36 square units is 6 units

C) The side length of a square pool with the area chosen in part A is 33•√33

What is a mathematical expression?

An expression is a mathematical statement which consists of 2 or more numbers and, or variables joined together by mathematical operators.

A) The given equations for the pool is presented as follows;

Area of the pool = ∆ × 102 + ∆ × 101 + 6

Area of the pool = x³

x = A whole number

Expressing the word problem mathematically gives;

∆ × 102 + ∆ × 101 + 6 = x³

203•∆ + 6 = x³

Which gives;

[tex] \displaystyle{ \triangle = \frac{ {x}^{3} - 6}{203} }[/tex]

Using a graphing calculator, when x = 33, we get;

[tex] \displaystyle{ \triangle = \frac{ {33}^{3} - 6}{203} = 177 }[/tex]

The value of the number is x = 33 and ∆ = 177

B) The perfect square that is closest to x = 33 is 36

The side length of a square pool with an area of 36 is √(36) = 6

C).The area of the pool chosen is 33³ = 35937

The side length of a square pool with an area of 35937 is √(35937) = 33•√(33)

Learn more about word problems in mathematics here:

https://brainly.com/question/2068175

#SPJ1

Please Help me solve this

Answers

The value of f'(x) at x = 1 is 1 for first order derivative of x^x^3.

A derivative in calculus is the rate of change of a quantity y with respect to another quantity x. It is also termed the differential coefficient of y with respect to x. Differentiation is the process of finding the derivative of a function.

Given that, x^x^3 and we have to find the first order derivative with respect to x and then find the value at x = 1.

Let's proceed to solve this question accordingly.

let f(x) = x^x^3

The first order derivative = f'(x) = d/dx(x^x^3)

First apply the generalized power rule, then we have

= x^x^3.d/dx(ln(x)x^3)

Applying the power rule, we get

= x^x^3 (d/dx(x^3).ln(x)+x^3.d/dx(ln(x)))

= x^x^3 (3x^2 ln(x) +x^3.1/x)

= x^x^3 (3x^2ln(x) +x^2)

On simplifying, we will get

= x^x^3+2(3ln(x)+1)

f'(x) = d/dx(x^x^3) = x^x^3+2(3ln(x)+1)

Now, at x = 1, we get

f'(x) = d/dx(x^x^3) = x^x^3+2(3ln(x)+1) = 1

Therefore, f'(x) = d/dx(x^x^3) = x^x^3+2(3ln(x)+1) = 1 is the required answer.

Learn more in depth about first order derivative at https://brainly.com/question/28376218

#SPJ1

Write a situation in which positive and negative numbers are used to describe values that have opposite meaning. What does 0 represent in this situation you created? (10 points)

Answers

A situation in which positive and negative numbers are used to describe values that have opposite meaning is that John is on top of a mountain that is 2000 above sea level and his friend is diving and is below 2000 feet.

The thing that 0 represents is the addition in the sea level.

How to illustrate the information?

Based on the information illustrated, the situation will be that John is on top of a mountain that is 2000 above sea level and his friend is diving and is below 2000 feet.

Therefore, the addition will be:

= 2000 + (-2000)

= 2000 - 2000

= 0

Learn more about numbers on:

brainly.com/question/148825

#SPJ1

List the angles of the triangle in order from largest to smallest.Question options:A) ∠A, ∠C, ∠BB) ∠C, ∠B, ∠AC) ∠B, ∠C, ∠AD) ∠A, ∠B, ∠C

Answers

Angles opposite to shorter sides are shorter. Then, the shortest angle is opposed to the shortest side.

Since the shortest side has a length 2.8 and is opposed to the angle A, then the shortes angle is A.

The next shortest side is that opposite to B, which has a length of 3.4.

And finally, the largest side has a length of 4.7 and it's opposite to the angle C.

Therefore, the list of angles from largest to smallest, is:

[tex]\angle C,\angle B,\angle A[/tex]

If a nonlinear system of equations contains one linear function that touches the quadratic function at its minimum, then the system has which of the following?
A. No solution
B. Infinitely many solutions
C. One solution
D. Two solutions

Answers

Answer:

  C.  One solution

Step-by-step explanation:

You want to know the number of solutions of a system of equation such that the graph of the linear function touches the graph of the quadratic function at its minimum.

Solutions

The number of real solutions of the system of equations is equal to the number of points of intersection of their graphs. The problem statement tells you the graphs intersect at one point, so there is one solution.

__

Additional comment

The linear function can only touch the minimum of a quadratic if its graph is a horizontal line.

<95141404393>

Taylor wants to use a scale factor of 12 to make a smaller drawing of the door image shown. Part of her work is shown. Finish her work to find the height and width of her scale drawing

Answers

the height and width of her scale drawing is respectively:

height = 1*1/2inch

width = 3/4 inch

what is dimension?

Dimension are the measure of the size or distance of an object or region or space in one direction.

so, all we needs to do is multiply is dimension of the original door image by the scale factor , to get the height and width of the scale drawing.

height:   3 inch x 1/2 =3x1/2=3/2=1x1/2 inch

width: 1x 1/2 inch x 1/2 =3/2 x 1/2= 3x1/2x2=3/4 inch

hence , the height and width of the scale drawing is respectively= 1x 1/2 inch ,3/4 inch

To know more about dimension click here

https://brainly.com/question/28835538

#SPJ9

For f(x) = 3x²-x+5, 2f(-3) - f(2)​

Answers

Answer:

55

Step-by-step explanation:

First solve for f(-3):

[tex]f(-3) = 3(-3)^2-(-3)+5[/tex]

[tex]f(-3)=3(9)+3+5[/tex]

[tex]f(-3)=27+3+5[/tex]

[tex]f(-3) = 35[/tex]

Second solve for f(2):

[tex]f(2) = 3(2)^2-(2)+5[/tex]

[tex]f(2) = 3(4)-(2)+5[/tex]

[tex]f(2) = 12-2+5[/tex]

[tex]f(2) = 15[/tex]

Now plug in f(-3) and f(2) to the expression:

[tex]2(f(-3))-(f(2)) =[/tex]

[tex]2(35)-(15)=[/tex]

[tex]70-15=55[/tex]

This is a step by step answer to your question in picture format. Hope this help.

write 7843 to nearest thousand​ with explanations

Answers

thoAnswer:8,000

Step-by-step explanation:
Because if you round to the nearest thousand 8 is above 5 so you round up making it 8000

In an experiment, college students were given either four quarters or a $1 bill and they could either keep the money or
spend it on gum. The results are summarized in the table. Complete parts (a) through (c) below.

Answers

Using the probability concept, we have that:

a) The probability is of 0.244.

b) The probability is of 0.756.

c) A student given a $1 bill is more likely to have kept the money.

What is a probability?

A probability is calculated as the number of desired outcomes in the experiment divided by the number of total outcomes in the experiment.

For item a, we have that out of 11 + 34 = 45 students who were given a $1 bill, 11 spent the money, hence the probability is given as follows:

p = 11/45 = 0.244.

For item b, we have that out of 11 + 34 = 45 students who were given a $1 bill, 34 kept the money, hence the probability is given as follows:

p = 34/45 = 0.756.

For item c, we have that a student given a $1 bill is more likely to have kept the money, as 0.756(kept) > 0.244(spent), which are the two probabilities we found in the previous items.

More can be learned about probabilities at https://brainly.com/question/14398287

#SPJ1

5. Geometry Audrey draws a triangle that has
these sides: s, s, and is. When the length of s
is doubled, the new perimeter is twice the old
perimeter less 14. What are lengths of
the triangle?

Answers

The length of each side is less than sum of the lengths of the other two sides and greater than the difference between these lengths.

2s is not less than 1/2s+ 2/3s

What is a triangle?

A triangle is polygon with the three edges and three vertices. It is one of the basic shapes in the geometry. A triangle with the vertices A, B, and C is denoted Δ ABC.

In the Euclidean geometry, any three points, when the non-collinear, determine a unique triangle and simultaneously, a unique plane (i.e. two-dimensional Euclidean space). In the other words, there is the only one plane that contains that triangle, and every triangle is contained in some plane. If the entire geometry is only  Euclidean plane, there is only one plane and all triangles are contained in it; however, in the higher-dimensional Euclidean spaces, this is no longer true. This article is about triangles in the Euclidean geometry, and in the particular, the Euclidean plane, except where otherwise noted.

new triangle perimeter is = 2s + 1/2s + 2/3s

This new triangle, however, cannot be created physically because

The length of each side is less than sum of the lengths of the other two sides and greater than the difference between these lengths.

2s is not less than 1/2s+ 2/3s

To know more about vertices visit: https://brainly.com/question/12516060

#SPJ9

Write an equation for a parabola with​ x-intercepts (-3,0) and (4,0) which passes through the point (2,-40)

Answers

The parabolic equation that passes through the points is y = 29/7x^2 + -27/7x - 342/7

What are parabolic equations?

Parabolic equations are second-order polynomial equations and they have the form y = ax^2 + bx + c or y = a(x - h)^2 + k

How to determine the equation of the parabola?

The given parameters are

x-intercepts (-3,0) and (4,0)

Point = (2,-40)

From the definition above, we have the form to be

y = ax^2 + bx + c

Substitute (-3,0) and (4,0)  in the above equation

So, we have

0 = a(-3)^2 + b(-3) + c

0 = a(4)^2 + b(4) + c

This gives

9a -3b + c = 0

16a + 4b + c = 0

Substitute (2,-40) in the above equation y = ax^2 + bx + c

a(2)^2 + b(2) + c = -40

So, we have

4a + 2b + c = -40

The equations become

9a -3b + c = 0

16a + 4b + c = 0

4a + 2b + c = -40

Using a graphing tool, we have

a = 29/7, b = -27/7 and c = -342/7

So, we have

y = ax^2 + bx + c

This gives

y = 29/7x^2 + -27/7x - 342/7

Hence, the equation is y = 29/7x^2 + -27/7x - 342/7

Read more about parabolic equation at

https://brainly.com/question/4061870

#SPJ1

how to find the measure of L

Answers

SOLUTION:

Step 1:

In this question, we are given the following:

Step 2:

he deatails of the solution are as follows:

rom tjhe triangle, we can see that Traingle KLM is an isosceles triangle, such that Angle J = 52.3 degrees suchn that:

[tex]Angle\text{ J = Angle K = 52. 3}^0(\text{ base angles are equal\rparen}[/tex]

Now, we have that:

[tex]\begin{gathered} Angle\text{ J +Angle K + Angle L = 180}^0 \\ 52.\text{ 3}^0+52.\text{ 3}^0+\text{ Angle L = 180}^0 \\ 104\text{. 6 }^0+\text{ Angle L = 180}^0 \\ Angl\text{e L = 180}^0-104\text{. 6}^0 \\ Angle\text{ L = 75.4}^0 \end{gathered}[/tex]

ONCLSUSION:

The measure of Angle L =

[tex]75.\text{ 4}^0[/tex]

Stella signed up for a streaming music service that costs $9 per month. The service allows Stella to listen to unlimited music, but if she wants to download songs for offline listening, the service charges $1.25 per song. How much total money would Stella have to pay in a month in which she downloaded 40 songs? How much would she have to pay if she downloaded ss songs?

Answers

The money would Stella have to pay in a month in which she downloaded 40 songs is $59.

The amount that she has to pay if she downloaded ss songs is 9 + 1.25s.

How to illustrate the information?

From the information, Stella signed up for a streaming music service that costs $9 per month and the service allows Stella to listen to unlimited music, but if she wants to download songs for offline listening, the service charges $1.25 per song.

The money would Stella have to pay in a month in which she downloaded 40 songs is:

= 9 + 1.25s

where s = number of songs

= 9 + 1.25s

= 9 + 1.25(40)

= 9 + 50

= $59.

The amount that she has to pay if she downloaded s songs is:

= 9 + 1.25(s)

= 9 + 1.25s

Learn more about music on:

brainly.com/question/26373912

#SPJ1

(11, 3) and (4, 3) on a coordinate plane?

Answers

The midpoint of the given points (11, 3) and (4, 3) on the coordinate plane is ( 15/2, 3 ).

How calculate the midpoint between two point?

A midpoint is simply a point that divides a line segment into two equal halves.

The midpoint formula is expressed as;

M = ( (x₁+x₂)/2, (y₁+y₂)/2 )

Given the data in the question;

Point 1( 11, 3 )

x₁ = 11y₁ = 3

Point 2( 4, 3 )

x₂ = 4y₂ = 3

Midpoint = ?

To find the midpoint, plug the given points into the midpoint formula above and simplify.

M = ( (x₁+x₂)/2, (y₁+y₂)/2 )

M = ( ( 11 + 4 )/2, ( 3 + 3 )/2 )

M = ( ( 15 )/2, ( 6 )/2 )

Midpoint M = ( 15/2, 3 )

Therefore, the midpoint of the given coordinates is ( 15/2, 3 ).

Learn more about the midpoint formula here: brainly.com/question/14687140

#SPJ1

Katie wants to save at least $280 to go on the math team field trip. He currently has $112 saved.
If he has 8 weeks left to save, which inequality and graph represent the amount of money per week he needs to save to meet the goal?

Answers

He must save 21 dollars per week to meet the goal.

How to find the amount to be saved?

Given,

Katie wants to save at least $280

He currently has $112 saved.

If he has 8 weeks left to save.

Solution:

The amount to be saved is $280 - $112 = $168

If he has 8 weeks left to save,

Per week he must save = $168/8

= $21

He must save 21 dollars per week to meet the goal.

To learn more about the amount, refer

https://brainly.com/question/25793394

#SPJ13

There are 28 students in a class.
16 of the students are girls.
What proportion are boys?
Write your answer in two ways.

Answers

Answer:

3:7

Step-by-step explanation:

Given:

There are 28 students in a class16 of the students are girls.

To Find:

What proportion of the students are boys.

Formula used:

No. of boys = Total students - No. of girls

Using the formula, we have

No. of boys = 28 - 16 = 12

Proportion of boys = 12/28 = 3/7

So, proportion of boys = 3:7.

An actor bought a​ pearl, diamond, and ruby necklace for his famous wife for $54,000. In 2011 this necklace was auctioned for $11,379,600. The auction price was what percent of the original purchase​ price?

Answers

This necklace sold at auction for $11,379,600 in 2011. The percentage of the initial purchase price was represented by the auction price is 2.1%.

Given that,

For $54,000, an actor purchased a pearl, diamond, and ruby necklace for his well-known wife. This necklace sold at auction for $11,379,600 in 2011.

We have to find what percentage of the initial purchase price was represented by the auction price.

Let us take the percentage as x%.

Necklace sold at auction=the percentage multiplied by the actual price

We get,

$11,379,600 =x%($54,000)

x%=$11,379,600/$54,000

x%=2.1%

Therefore, the percentage of the initial purchase price was represented by the auction price is 2.1%.

To learn more about auction visit: https://brainly.com/question/24261307

#SPJ1

b(x)=18-0.5x 6 and 18

Answers

The solution of B(x) = 18 - 0.5x with x being -2, 0, 5 is 19, 18 and 15.5.

How to evaluate functions?Finding the value of f(x) =... or y =... that corresponds to a certain value of x is what it means to evaluate a function. Simply swap out all instances of x with the value of x to do this. For example, if we are requested to evaluate f(4), then the value 4 has been given to x.

Given:

B(x) = 18-0.5x

x = -2, 0, 5

Putting the values of x to find the result of the function.

B(-2) = 18 - 0.5 × (-2) = 18 + 1 = 19

The required solution of the function at x = -2 is f(-2) = 19.

B(0) = 18 - 0.5 × (0) = 18

The required solution of the function at x = 0 is f(0) = 18.

B(5) = 18 - 0.5 × (5) = 18 - 2.5 = 15.5

Thus, the required solution of the function at x = 5 is f(5) = 15.5.

To learn more about evaluating functions visit:

https://brainly.com/question/14311853

#SPJ9

The complete question is :"How do you solve B(x) = 18-0.5x with x being -2, 0, 5."

help pls!! In the figure below, N is between M and O, and O is between N and P. If NO=2, NP = 8, and MP=15, find MO.

Answers

The distance between MO is 9.

How to find MO ?

From the question MP is straight line and O & N is points between M & P.

No is 2NP is 8MP is 15

so the total length of line is 15

Need to find MO , To find  MO = MP - (NP - NO)

                                                   = 15 - (8 -2)

                                                  = 15 - 6 = 9

The MO is 9.

To find points in graph :

To find a line that's parallel to a line and goes through a particular point, use the point's coordinates for (x1, y1) in point slope form: y - y1 = m (x - x1).The slope intercept formula y = mx + b is used when you know the slope of the line to be examined.Use the slope and one of the points to solve for the y-intercept.One of your points can replace the x and y, and the slope you just calculated replaces the m of your equation y = mx + b. Then b is the only variable left. Use the tools you know for solving for a variable to solve for b.

To learn more about finding distance refer :

https://brainly.com/question/17273444

#SPJ13

Write the fractions in order from least to greatest: 2/9, 1/6, 1/5, 3/10

Answers

Answer:

1/5,1/6,2/9,3/10

Step-by-step explanation:

hope it helps helps and have a nice day! :)

15) A sample of 4 different calculators IS randomly selected from a group
containing 46 that are defective and 26 that have no defects. What is the
probability that all four of the
calculators selected are defective? Round to four
decimal places.
A) 0.1021
B) 0.1586
C) 0.1666
D) 10.9154

Answers

Answer:

C) 0.1666

Step-by-step explanation:

The probability of selecting a calculator that is defective can be defined as: [tex]\frac{46}{46+26}[/tex]

The 46 in the numerator is the number of calculators that are defective in the group. The 46 + 26 represents the total amount of calculators since 46 are defective and 26 are not defective and assuming a calculator can only be defective or not defective then these are the total number of calculators in the group.

This gives you a probability of approximately: [tex]\frac{46}{72}[/tex] or approximately 0.638889

We can multiply independent events to find the combined probability of these events occurring. So we have to assume you put the calculator back into group after selecting one.

If this is the case we simply multiply 46/72 * 46/72 * 46/72 * 46/72 or (46/72)^4 to get an approximate probability of: 0.1666

For the function ​f(x)=4x+4​,
find ​(a)​ f(x+​h),
​(b)​ f(x+​h)−​f(x), and ​
(c) f(x+h)−f(x) h.

​(a)​ f(x+​h)=?

Answers

Step-by-step explanation:

a)

[tex]f(x+h) = 4(x+h)+4[/tex]

[tex]f(x+h)=4x+4h+4[/tex]

b)

[tex]f(x+h)-f(x)=4x+4h+4-(4x+4)[/tex]

[tex]f(x+h)-f(x)=4x+4h+4-4x-4[/tex]

[tex]f(x+h)-f(x)=4h[/tex]

c)

[tex]f(x+h)-f(x)h=4x+4h+4-h(4x+4)[/tex]

[tex]f(x+h)-f(x)h=4x+4h+4-4hx+4h[/tex]

[tex]f(x+h)-f(x)h=-4hx+8h+4x+4[/tex]

3. John and Savanah are saving money to go on a trip to Mexico. They need at least $2,545 in order to go. John tutors English and Savanah works as a babysitter to raise money. John charges $15 per hour and Savanah charges $20 per hour. The number of hours that Savanah has scheduled is no more than five times the number of hours John has scheduled. Savanah will babysit at least 40 hours.. Write a set of constraints to model the problem, with x representing the number of hours John tutors and y representing the number of hours Savanah babysits. Answer:​

Answers

The group of restrictions used to simulate the issue includes

y<5x andy [tex]\geq[/tex] 40hours

How to construct a set of restrictions to represent the issue:

The issue includes the following information.

They need a minimum of $2,545 divided by the hours John instructs and the hours Savanah watches children.There is a maximum of a five-fold difference between the number of hours Savanah has booked and that John has.At least 40 hours will be spent watching the children by Savanah.

The following are the restrictions for modeling the issue: There is a maximum of a five-fold difference between the number of hours Savanah has booked and that John has.

Savanah will provide childcare for at least 40 hours, with no more than meaning it is not larger than that which suggests it is less than, Hence, y<5x

Meaning "at least 40 hours" is "at least 40 hours." 40 hours or more may be expressed as

y ≥ 40 hours

The two necessary restrictions are represented as y< 5x and y=> 40 hours in an inequality model.

Constraints are discussed in greater detail at https://brainly.com/question/17156848

#SPJ1

Find the x-intercept of the function g(x)=8x²-10x-3

Answers

To find the x-intercepts of the function g(x), we must set it equal to 0 and solve for x.

We have the following:

[tex]\begin{gathered} g(x)=8x^2-10x-3 \\ if\text{ g(x)=0} \\ \Rightarrow8x^2-10x-3=0 \end{gathered}[/tex]

we can use the quadratic formula to get the roots of the polynomial:

[tex]\begin{gathered} a=8 \\ b=-10 \\ c=-3 \\ x_{1,2}=\frac{-b\pm\sqrt[]{b^2-4ac}}{2a} \\ \Rightarrow x_{1,2}=\frac{-(-10)\pm\sqrt[]{(-10)^2-4(8)(-3)}}{2(8)}=\frac{10\pm\sqrt[]{196}}{16} \\ \Rightarrow x_{1.2}=\frac{10\pm14}{16} \\ \Rightarrow x_1=\frac{10+14}{16}=\frac{24}{16}=\frac{3}{4} \\ \Rightarrow x_2=\frac{10-14}{16}=\frac{-4}{16}=-\frac{1}{4} \end{gathered}[/tex]

therefore, the x-intercepts of the function g(x) are the points (3/4,0) and (-1/4,0)

-8>-3t-10 can someone help

Answers

Answer:

t > -2/3

Step-by-step explanation:

−8>−3t−10

Step 1: Flip the equation.

−3t−10<−8

Step 2: Add 10 to both sides.

−3t−10+10<−8+10

−3t<2

Step 3: Divide both sides by -3.

−3t/−3 < 2/−3

t > −2/3

Members of the marching band line up in 6 rows of 8.

Answers

Answer: 6x8

Ur answers should be 48 and again we did repeated addition so

6+6+6+6+6+6+6+6 or we can do 12+12+12+12 or 24+24 which we should know use 48

Step-by-step explanation:

Find the center, transverse axis, vertices, foci, and asymptotes. Graph the equation.y² -x² =25
What are the asymptotes?
The asymptote with positive slope is, and the asymptote with negative slope is.

Answers

The values are as follows:

center:  y²/25-x²/25 = 1

transverse axis : b = 5

Vertices:  (0,5) and (0,-5)

Foci: (0,5√2) and (0,-5√2)

Asymptotes: y = x and y = -x

An asymptote is a line being approached by a curve but never touching the curve. i.e., an asymptote is a line to which the graph of a function converges. We usually do not need to draw asymptotes while graphing functions.

Types of Asymptotes:

1) Horizontal asymptote (HA) - It is a horizontal line and hence its equation is of the form y = k.

2) Vertical asymptote (VA) - It is a vertical line and hence its equation is of the form x = k.

3) Slanting asymptote (Oblique asymptote) - It is a slanting line and hence its equation is of the form y = mx + b.

y² -x² =25 up down hyperbola with (h,k) = (0,0) a = 5 and b = 5

(y-k)²/a²-(x-h)²/b² = 1 is the standard equation of up-down hyperbola.

With semi-axis is a and semi-conjugate axis is b.

and centre (h,k).

Center = (y-0)²/5²-(y-0)²/5² = 1

= y²/25-x²/25 = 1

The axes are a = 5 and b = 5

transverse axis is b = 5

The foci are given by (h, k+c) and (h, k-c), where c = √a²+b²

c = √5²+5² = 5√2

therefore, the foci are (0,5√2) and (0,-5√2).

The vertices are (h, k+a) and (h, k-a)

(h,k) = (0,0) and a = 5 and b = 5

vertices are (0,5) and (0,-5)

Asymptotes are y = x and y = -x.

Therefore, the summary is as follows:

center:  y²/25-x²/25 = 1

transverse axis : b = 5

Vertices:  (0,5) and (0,-5)

Foci: (0,5√2) and (0,-5√2)

Asymptotes: y = x and y = -x

Learn more in depth about similar problem at https://brainly.com/question/4138300

#SPJ1

Find the direction angle of v for the following vector.v=−6i−2jWhat is the direction angle of v?

Answers

Given:

v is the given vector.

[tex]v=-6i-2j[/tex]

To find:

Find the direction angle of v.

Formula to find the direction:

[tex]\tan \theta=\frac{y}{x}[/tex]

From the given vector x and y are:

[tex]x=-6\text{ \& y=-2}[/tex][tex]\begin{gathered} \tan \theta=\frac{-2}{-6} \\ \theta=\tan ^{-1}(\frac{1}{3}) \\ \theta=18.4\degree \end{gathered}[/tex]

please help me to do this problem.

Answers

[tex]V\text{ = }\frac{1}{3}Bh[/tex]

3 is dividing on the right, then it will multiply on the left

B is multiplying on the right, then it will divide on the left

[tex]\frac{V\cdot3}{B}=h[/tex]

Other Questions
You are running a fuel economy study. One of the cars you find is blue. It can travel 31 1/2 miles on 1 1/4 gallons of gasoline. Another car is red. It can travel 21 3/5 miles on 4/5 gallons of gasoline. What is the unit rate for miles per gallon for each car? Which car could travel the greater distance on 1 gallon of gasoline?ok nevermind I found the answerThe unit rate for the blue car is 25 1/5The unit rate for the red car is 27The red car could travel a greater distance on 1 gallon of gasoline Slope intercept form for points through (0,-5) and (4,0) Which of the following amino acids in a protein are likely to be buried deeply in a protein away from water when folded into their tertiary conformation: A. Serine, threonine, glutamine B. Lysine, arginine, histidine C. Valine, leucine, isoleucine D. All of the above E. None of the above SernaAll changes saved3. What stroke of an engine moves the piston down into the engine and causes the top ofcylinder to open and mix air and fuel? if RS equals 59 and ST equals 10x - 31 find x The double number lines show the ratio of yards to miles.How many yards are in 4 miles? What is the value of x? Complete all problems (there is one on1. Let f(x)=x and let g(x) = 2x - 8a. Find fo g(x) and its domain and rangeb. gof(x) and its domain and rangec. What is go f (3)? Dividing Complex Numbers: Simplify 5/-5i The shortest side of a right angle triangle is 6cm shorter than the hypotenuse. The difference in length of the other two sides is 3cm. If the shortest side is n - 3, show that 2n squared equals 12n. Hence, n. I need help because i cant seem to understand it A block of wood has a mass of 120 gand a volume of 200 cm. What is the density ofthe wood? Lucy used 3 3/4 cups of sugar for her lemonade stand. She divided thesugar, in equal amounts, into 6 pitchers. How much sugar went into eachpitcher? Which statement is true regarding the graphed functions?On a coordinate plane, a straight red line with a positive slope, labeled g of x, crosses the x-axis at (negative 6, 0) and the y-axis at (0, 6). A straight blue line with a negative slope, labeled f of x, crosses the x-axis at (negative 0.75, 0) and the y-axis at (0, negative 2). Which of the following is NOT a way to change a rhythm?A. Adding syncopationB. Adding sixteenth notesC. Adding an ostinatoD. Adding a dotted rhythm A department store purchases screen-printed t-shirts at a cost of $5 per shirt. they mark up the price 150% (making the selling price 250% of the store's purchase price) and put them on the sales floor. every month that a t-shirt doesn't sell, the store reduces the selling price by 25%. which expression shows the selling price after one monthly price reduction? $5 $5(1.50) " $5(0.25) $5 ($5)(0.25) " ($5)(1.50) $5(2.50) " $5(0.25) ($5)(2.50) " ($5)(2.50)(0.25) A(c-b)=dSolve for cPlease and thank Please help me!! (Only answer if you know it) thanks!Read stanza 2 of the poem. How does Poe use hyperbole to develop a theme in the poem?I was a child and she was a child,In this kingdom by the sea;But we lived with a love that was more than love,I and my Annabel Lee;With a love that the winged seraphs of the heaven Coveted her and meIn this stanza, Poe uses hyperbole _____________and__________. This builds on the idea that the speaker is _______.Options for the first drop-down menu: A.) to create a vivid image for the readerB.) to exaggerate or emphasize or overemphasize the speakers love for Annabelle LesSecond option for drop down menu: A.) to blame someone or something for her deathB.) to highlight how rare the speakers relationship wasThird option for drop-down menu: A.) a caring parter B.) overcome by grief Which of the following functions is graphed below? An investor purchased 50 shares ofstock in a company for $20 pershare. One year later, the investorsold all the shares for $925. What isthe investor's rate of return?A. 7.5%C. -7.5%B. -8.1%D. 8.1%